Q9

 
hyperblue6
Thanks Received: 0
Vinny Gambini
Vinny Gambini
 
Posts: 3
Joined: February 09th, 2013
 
 
 

Q9

by hyperblue6 Tue Aug 27, 2013 12:49 pm

I went through all the choices and I was able to eliminate all of them. I think I'm getting confused as to what is the complete and accurate list. Can anyone help me unlock this trick? How is E the correct answer?
 
christine.defenbaugh
Thanks Received: 585
Atticus Finch
Atticus Finch
 
Posts: 536
Joined: May 17th, 2013
 
This post thanked 1 time.
 
 

Re: Q9

by christine.defenbaugh Wed Aug 28, 2013 3:04 am

Complete and accurate list questions can be tricky, if we're not very clear about what kind of list we are looking for.

In this question, our task is to identify the complete and accurate list of all the medicines that can be ranked fifth. In other words, we're looking for a list of EVERY medicine that can be fifth (complete), and ONLY those medicines that can be fifth (accurate). Since we have two frames, we have to include anything that could ever be ranked fifth in either frame.

Image

Who can never be fifth? L must be second in both frames, so it's never fifth. Eliminate (C). K must come before F in Frame #2, and so cannot be fifth there. In Frame #1, K is forced out. So it cannot be fifth in either frame. Eliminate (D).

Of our remaining answer choices, (A), (B), and (E), all three contain G, H, and M. So we know those can be fifth in at least one frame. So we only need to test F and I. F can't be fifth in Frame #1, so let's use Frame #2 to test:

F works just fine as fifth in the lineup: G-L-I-K-F (H, M out)
I works just fine as fifth in the lineup: G-L-K-F-I (H, M out)

So, the complete and accurate list should contain both F and I. (E) is correct.

If you got turned around on what the question was asking, and instead thought that you needed a list of the medicines that could be fifth in BOTH frames, then you would have eliminated all the answer choices! That list would have been far shorter. (The only one who could be fifth in both frames is I.)

Please let me know if that answers your question!
 
amil91
Thanks Received: 5
Elle Woods
Elle Woods
 
Posts: 59
Joined: August 02nd, 2013
 
 
 

Re: Q9

by amil91 Thu Oct 17, 2013 10:09 pm

christine.defenbaugh Wrote:Complete and accurate list questions can be tricky, if we're not very clear about what kind of list we are looking for.

In this question, our task is to identify the complete and accurate list of all the medicines that can be ranked fifth. In other words, we're looking for a list of EVERY medicine that can be fifth (complete), and ONLY those medicines that can be fifth (accurate). Since we have two frames, we have to include anything that could ever be ranked fifth in either frame.

Image

Who can never be fifth? L must be second in both frames, so it's never fifth. Eliminate (C). K must come before F in Frame #2, and so cannot be fifth there. In Frame #1, K is forced out. So it cannot be fifth in either frame. Eliminate (D).

Of our remaining answer choices, (A), (B), and (E), all three contain G, H, and M. So we know those can be fifth in at least one frame. So we only need to test F and I. F can't be fifth in Frame #1, so let's use Frame #2 to test:

F works just fine as fifth in the lineup: G-L-I-K-F (H, M out)
I works just fine as fifth in the lineup: G-L-K-F-I (H, M out)

So, the complete and accurate list should contain both F and I. (E) is correct.

If you got turned around on what the question was asking, and instead thought that you needed a list of the medicines that could be fifth in BOTH frames, then you would have eliminated all the answer choices! That list would have been far shorter. (The only one who could be fifth in both frames is I.)

Please let me know if that answers your question!

While this explanation is excellent, I figured I would add the way I did it as it was a bit different, but was easy to do and worked for me since I did not frame this question.

I used my previous work to figure out which letters could be last. If you look at question 6 the correct answer has M last. For question 7, the correct answer has I before F and a hypothetical I had to get me to that had F last. So from these two questions I knew the correct answer for question 9 had to F and M in it. So with this information I could eliminate answers B, C, and D since they do not have both F and M. From that point I used Christine's strategy to compare the differences of the answers; the only difference in the answer choices left, A and E, is the letter I. I then did a quick hypothetical to see if I could be last and It could. This probably wasn't the most efficient way to answer this question as I now have seen that framing it would have made things quite simpler, but while doing it I didn't think to frame and that's how I decided to attack the problem.
Last edited by amil91 on Fri Oct 25, 2013 8:43 pm, edited 1 time in total.
User avatar
 
tommywallach
Thanks Received: 468
Atticus Finch
Atticus Finch
 
Posts: 1041
Joined: August 11th, 2009
 
 
 

Re: Q9

by tommywallach Mon Oct 21, 2013 11:48 am

Great method, Amil! I'm a big fan of framing when there are only two possibilities, but you always have the option to "not" frame. It requires more inferences on questions, but fewer up top.

Good luck!

-t
Tommy Wallach
Manhattan LSAT Instructor
twallach@manhattanprep.com
Image